LSAT and Law School Admissions Forum

Get expert LSAT preparation and law school admissions advice from PowerScore Test Preparation.

User avatar
 Dave Killoran
PowerScore Staff
  • PowerScore Staff
  • Posts: 5853
  • Joined: Mar 25, 2011
|
#87686
Complete Question Explanation
(The complete setup for this game can be found here: lsat/viewtopic.php?f=151&t=8310)

The correct answer choice is (B)

If S departs before P, we can add in our knowledge that P departs earlier than R to create a sequence governing three of the four unassigned planes:

G3-Q15.png

A quick glance at the answer reveals that (B) is the most promising answer in this Cannot Be True question:

As S is at the beginning of the sequence above, S departing first is unlikely to be a problem. Thus, answer choice (A) is less attractive than (B), which also contains S.
As T is not addressed within this sequence, and T has no domestic or international assignment in the rules, T is an unlikely plane to figure in the correct answer choice. Thus, answer choices (C), (D), and (E) are all unlikely to be the correct answer.
Hence, answer choice (B) is the most likely correct answer based on a brief survey of the five answers. Let us examine (B) in more detail.

If S departs third, then from the sequence above P must depart fourth, and R must depart last. But, as ascertained during the setup, Q must depart last. Consequently, within this question S cannot depart third, and answer choice (B) cannot be true and is correct.
You do not have the required permissions to view the files attached to this post.

Get the most out of your LSAT Prep Plus subscription.

Analyze and track your performance with our Testing and Analytics Package.